Overunity.com Archives is Temporarily on Read Mode Only!



Free Energy will change the World - Free Energy will stop Climate Change - Free Energy will give us hope
and we will not surrender until free energy will be enabled all over the world, to power planes, cars, ships and trains.
Free energy will help the poor to become independent of needing expensive fuels.
So all in all Free energy will bring far more peace to the world than any other invention has already brought to the world.
Those beautiful words were written by Stefan Hartmann/Owner/Admin at overunity.com
Unfortunately now, Stefan Hartmann is very ill and He needs our help
Stefan wanted that I have all these massive data to get it back online
even being as ill as Stefan is, he transferred all databases and folders
that without his help, this Forum Archives would have never been published here
so, please, as the Webmaster and Creator of this Forum, I am asking that you help him
by making a donation on the Paypal Button above
Thanks to ALL for your help!!


A possible violation of the Law of Conservation of Energy

Started by Zetetic, April 14, 2015, 04:59:57 PM

Previous topic - Next topic

0 Members and 1 Guest are viewing this topic.

ayeaye

I just did experiments, to see what happens. Yes i also tried to make a full circle of magnets but this, maybe indeed became what you say "unmagnetized". That is, the poles of some magnets shifted, for some unknown reason. And this is why the continuous rotation was not possible.

The experiments showed overunity only with shorter chains of magnets, something changed whenever it was made to full circle.

My hand, all you have to do is to move the disc near the top of the peak near the first magnet. You feel it, and you have to move it as close to that peak as possible. Then it moves in different directions depending on to what side of the peak you moved it. Your hand of course provides energy, but if you move it near the peak every time, the energy you provided should be the same both when the disc moves in one direction, or in the direction opposite to it. Now what shows overunity is that it moves some two times more in one direction, than in the opposite direction, in spite that the initial energy you provided was the same.

I did it with my hand, you can try it with your hand, and if the magnets and the disc are well repeated, you should get the same results. I know that its better to use mechanical things, and not hand, as hand is not very reliable, and the video cannot show all the slight movements of the hand, which could affect the result. But the experiment is simpler and thus much more easily repeatable that way. Because what i also know is that no experiment can be provable, experiments can only be repeatable. Thus easy repeatability is better than more provability. So i made the experiment as simple and as easily repeatable as possible.

I used a computer's processor fan for bearings, but i oiled it well. And i used only mounting tape for fixing things. Tesa, scotch, 3m, whatever you get works well. But magnets are a bigger problem, i don't know where to get so small ceramic magnets which i used in the video 3/4. I got them from some small decorations which they put on the refrigerator door, but not all of them have so small magnets. One may try very small neodymium cube magnets, which can be bought, but i'm not sure whether the result will be the same.

What concerns the law of conservation of energy, then i think it is in the very essence just an expression of the balance in the universe. So even if you can show that it is not valid in one process, it is valid more generally. But no matter what the theories, conservation of energy or the energy came from some unknown source, i don't want to speculate where the energy came from. Because if i did, and if it happens that my guess was wrong, then for that same reason it would be considered that all that i said was wrong.

Zetetic


ayeaye,



Thank you for continuing the conversation on Conservation of Energy.

"So even if you can show that it is not valid in one process, it is valid more generally."    -ayeaye

This is the crux of the issue.

The Law of Conservation of Energy is a "Law" and not a "Principle."

Mainstream Physics does not claim that energy is "generally" conserved.  It claims that energy is always conserved.  They claim that no one can ever "show that it is not valid in one process."

If they are right, then all the attempts at creating an Overunity Device as seen in this forum are nothing more than a waste of time.  A successful Overunity Device would be a mechanical device that violates the Law of Conservation of Energy, and if they are right, then no Overunity Device attempt will ever succeed.

This is the issue.  And this is the point of me posting a link to my disproof of this "Law" in this forum.

And so it's my hope and you, or anyone else in this forum who disagrees with mainstream Physics and believes an Overunity Device is possible (and therefore necessarily believes the "Law of Conservation of Energy" is not a "Law" but a "Principle"), might then discuss with me whether or not my argument (linked to in the OP) does actually disprove this "Law."

That's it.  That's my purpose in posting here.


---


I think I've figured out why you get the results you get in "Field lines chain motor 3/4."


Think of a hypothetical frictionless pendulum (see Q in the drawing below).

The pendulum starts out at rest and at a certain height (Q.1).  It then swings down and increases in velocity until it reaches its lowest point (Q.2).  And then it swings back up and decelerates until it finally comes to a stop and at the same height from which it started (Q.3).

The velocity reached at its lowest point is the exact amount of speed needed for it to then reach the same height on the other side (again, in a hypothetical frictionless world).

The same thing can be done with a magnet on a rotating disk and an external fixed magnet (see R in the drawing below).  Again, this is in a hypothetically frictionless world.  The two magnets (the moveable one and the fixed one) are orientated so that their dissimilar poles, which are attracted to one another, will pass by one another facing one another.

The moveable magnet starts at a rest at a certain distance from the fixed magnet (R.1).  It then rotates towards the fixed magnet and increases in velocity until it reaches the fixed magnet (R.2).  And then it continues to rotate beyond the fixed magnet and decelerates until it finally comes to a stop at the same distance from the fixed magnet from which it started (R.3).

The velocity reached when it is at the point of the fixed magnet is the exact amount of speed needed for it to then reach the same distance from the fixed magnet on the other side (in a hypothetical frictionless world).

Now the same thing is done again, but with the magnets orientated relative to one another as they are in "Field lines chain motor 3/4."

The moveable magnet starts out at rest just to the left of the fixed magnet where it will be repulsed to the right and not to the left (S.1).  But at this point, and this is the key point, the moveable magnet is both repulsed and attracted to the fixed magnet.  In this position, the right end North Pole of the moveable magnet is repulsed by the North Pole of the fixed magnet and the left end South Pole of the moveable magnet is attracted to the North Pole of the fixed magnet.  The moveable magnet then rotates to the left and increases in velocity until it reaches the point of greatest attraction between the moveable and fixed magnets (S.2).  It increases in velocity due to both repulsion and attraction.  It then continues to rotate beyond the point of greatest attraction and decelerates due to the attraction between the moveable magnet and the fixed magnet.  It decelerates, and again this is part of the key point, due now only to attraction (S.3).

The velocity it reaches when at the point of greatest attraction is from repulsion and attraction and so will be greater than the velocity needed to escape and move beyond the attraction (and only the attraction) while it is decelerating on the other side.

This is why, I think, you get the results you do in 3/4.  (And, even though you've taken steps to reduce the friction as much as possible, in your real world model (as in all real world models) there is friction and this is my guess why it then finally does come to a stop after escaping the attraction with the fixed magnet.)

So, you and I may disagree about the reason why for the results you get.  I hope I haven't offended you with an alternative theory.  Maybe I'm wrong.  But this is what makes sense to me right now as I look at it.  I'm happy to continue to discuss it with you and even to try to find flaws in my analysis!  (I'm interested in truth in the end and I'm not interested in "winning" arguments in the passing.)

Again, I hope you don't mind me offering an alternative theory.

Take care!

- Zet




(PS:  I've never built your exact model in "3/4" but I've built similar kinds of models.  I, too, have "felt" the "peak" of the magnetic repulsion.  I've always found this a fun and fascinating experience!)





ayeaye

My drawing was about one magnet in a chain of magnets, and i have only seen overunity with a chain of magnets. Ultimately the friction stops the movement of course. But overunity is because the repulsion of the first magnet of the chain should be the greatest propulsion there, if the other magnets in the chain do not add additional energy. But because the disc moves some two times more by the chain, they evidently do, the way the drawing shows.

The energy necessary to move the disc to the peak of the magnetic force at the first magnet in the chain, is also equal to the energy released by the repulsion of that magnet. When the disc moves some two times more than that by the chain of magnets, the energy released is more than the input energy. The known input energy that is, because the only known input energy is the energy necessary for moving the disc to the peak of the magnetic force.

allcanadian

@Zetetic
Quote
If any of you have ever posted anything on Science Forums you know that they love to make snarky comments putting people like you and me in our place
when we question the "known Laws of Physics." Their silence is, I believe, very telling.  It makes me think that I may really be onto something.  I may be wrong (I am all the time), but right now I think I really may have found something.
Again, I don't think it leads the mechanics needed to make an actual free energy device.  But, maybe it does and one of you can find it.  I don't know.
You make a very good case and I had to think on it for a while. I believe the conservation of energy relates to the known laws of physics however one can never know all of the near infinite number of ways the law(s) may be applied.
You know within a few minutes of reading your post I may have found a way to prove the paradox. Now imagine we had a very strong permanent magnet insulated from a hot metal plate in a box by a near perfect insulator such as aerogel. If we had metal balls inside the box then they would be attracted upward to the magnet however they would first impact the metal plate generating heat and this heat would partially demagnetize them causing them to fall to the bottom and repeat the cycle. Obviously the metal plate must get hotter from these impacts regardless of whether it was preheated to start the process or not. As well the impact in itself is also know to disturb the magnetic domains only increasing the effect. We could also pull a vacuum in the box to reduce the heat loss from top to bottom. Now if the bottom of the box dissipated part of the heat generated by the impacts cooling the balls just below the Curie temperature then we would theoretically have Maxwells demon.

The metal balls would in effect be the Demon generating heat through the transformation of kinetic energy to thermal energy on impact which in effect maintains their motion. However we should note the transformation is simply the macroscopic kinetic energy reduced to kinetic energy acting on a smaller scale we call heat. This also relates to a statment by Faraday-- "It does not matter how the change occurs only that it does". At which point we might ask would this be overunity? Well no because the universal radiation field induced the charge on the fundamental particles in matter which caused them to move which created a displacement field we call magnetism which was then disturbed by the thermal energy created on impact. The supposed extra energy would still be real energy transferred from the external field to make up the difference it's just that we never thought of that. Overunity is simply a convienient term to describe the fact we don't understand what the hell is going on, lol.
Would this ill concieved conservation breaking demon machine work in reality?, most likely not however is does raise some interesting questions.
AC

Knowledge without Use and Expression is a vain thing, bringing no good to its possessor, or to the race.

Zetetic



allcanadian,



Thank you for reading my argument against (hopefully my "disproof of") the Law of Conservation of Energy.

-

"I believe the conservation of energy relates to the known laws of physics however one can never know all of the near infinite number of ways the law(s) may be applied." – allcanadian

Are you saying that the Law of Conservation of Energy is generally true but not absolutely and always true (as ayeaye seemed to suggest above)?  If so, I agree with both of you guys.  But this means we are all in disagreement with mainstream physics.  And that's the point of my argument in Science Forums.

My argument (linked to in the OP) is a very simple piece of logic that (hopefully) shows that there is a flaw in the logic of the Law of Conservation of Energy.  And this means we can know that this "Law" is false.  And we can know that it is false to a logical certainty!  (No experiments needed.)

And this, I would think, would be something of interest to people who are trying to build mechanical devices where more energy is outputted than is imputed (which, if successful, would be a violation of the Law of Conservation of Energy, and where, alternatively, if this "Law" is in fact true, then these devices could never really work).

That's it.

(If anyone does read my argument linked to in the OP, please feel free to just read the question and ignore or skim over the endnotes.  The question is very straight forward and simple, but the endnotes, at times, are a bit tedious.  The endnotes are there simply in anticipation of the likely issues to be raised in response to my argument.  They are superfluous.)

-

Before I comment on your two balls in a box example, I want to make sure I understand the set up.  In drawing M below I have visually described what I think you are saying.  There's a good chance I'm wrong, so please correct me if so.  That's the point of me drawing it out: to make sure I understand your set up.  And then, once we are on the same page, I'd love to comment on it and discuss it with you.

However, please note (as I'm sure you know) that Maxwell's Demon is a Second Law of Thermodynamics issue and not a Law of Conservation of Energy issue.  Both "Laws," and the hopeful disproof of both "Laws," are relevant to people like us who want to see successful perpetual motion machines.  But they are two different and distinct theories, and the mechanical devices that purport to violate them are two different and distinct kinds of mechanical devices.

If my argument linked to in the OP tends to inspire a Second Law of Thermodynamics analysis then, my bad, it should not have.  There is a "heat pack" in that argument.  But it is only there for the purpose of demagnetizing (raising above the Curie temperature) the magnets, and therefore ending the mutual attraction between the two magnets.  The thermal energy generated does nothing else in that thought experiment.  It does not create more or less (macro) kinetic energy.  My argument in the link in the OP is a disproof (hopefully) of the Law of Conservation of Energy and not an attempted disproof of the Second Law of Thermodynamics (although I suspect the latter must also have exceptions, just not in the argument I made, too).

I hope that's clear.  If not, please let me know.

So, while I suspect your two balls in a box example is beyond the scope of what I originally posted in this thread, I'm interested.  I'd like to understand it and discuss it.  So, please, review my drawings below and see if I've understood or not your proposal.  And, then, we can talk about the specifics of it.

Take care!

---


ayeaye,


I understand that in your "3/4" you have a row (a cured row) of multiple magnets and in my drawing S in Reply #11 I show only one magnet.  But I think the dynamics would be just the same.

Consider, for the ease of analysis, a straight magnet and a straight row of magnets with no gaps and a straight row of magnets with gaps (drawing N below).

In all three cases, the greatest point is repulsion is on the right end and the greatest point of attraction is on the left end.  In the case of the single bar magnet there are no other points of repulsion or attraction in between.  In the case of row of magnets with no gaps, theoretically, there are no points of repulsion or attraction in between (if all the magnets are perfectly aligned, but in the real world there will always be some physical imperfections and so there will be some (very minor) repulsions and attractions in between).  And in the case of the row of magnets with gaps there will definitely be points of repulsion and attraction in between, but these will be minor relative to the repulsion at the right end of the row and the attraction at the left end of the row.

(I make the claims that I do in the above paragraph based on years and years of playing around with magnetic configurations like this in an attempt to find a dynamic that would then lead to a successful perpetual motion machine.  I never did.)

If I understand you correctly, it is your proposal that with the gaps in in "3/4" each individual magnet in the row gives a separate push on the disk as they pass by the external fixed magnet.  It is my contention that the gaps, while they do have some impact on the system, do not push the disk further along.  It is my contention that whether the gaps are present or whether you did the same thing with one magnets or with a row of magnets with no gaps, you would get the same results as you do with the gaped row (curved gaped row) of magnets in "3/4."  It is my contention that if you have one magnet the same (or similar) length as the row of gapped magnets that you'd get the same (or similar) distance clearance in the end in both cases (and that it is not the individual separated magnets that each give the moving body a separate and additional push, but rather the repulsion and attraction at the right and left ends of both systems).

Again, I hope you don't mind me offering an alternative theory for your results in "3/4."  We are on, and beyond, the borderlines of accepted science and so we should expect (just as there is within the borderlines of accepted science) lots of disagreements!  I love your design.  I never tried building your exact model.  But I did work with similar ideas.  (And perhaps you can find the exception, if there is one, in this area that I was never able to find!)

Take care.

---


- Zet